Wave Equation with initial conditions, boundary condtions

Click For Summary
The discussion focuses on solving the wave equation using D'Alembert's formula, particularly under the constraints of odd initial conditions for functions f and g. The user expresses uncertainty about extending these initial conditions to ensure that f(-x) = -f(x) and g(-x) = -g(x). They propose defining new functions to create odd extensions but seek clarification on the correct approach. The conversation emphasizes the importance of correctly applying D'Alembert's solution and the need for further exploration of the initial conditions. Participants are encouraged to share their solutions or insights on the topic.
mmmboh
Messages
401
Reaction score
0
ojfq7p.jpg


So, I do not think I did this properly, but if f(-x)=-f(x), then u(-x,0)=-u(x,0), and if g(-x)=-g(x), then ut(-x,0)=-ut(x,0).

According to D`Alambert`s formula,

u(x,t)=[f(x+t)+f(x-t)]/2 + 0.5∫g(s)ds (from x-t to x+t)
so, u(0,t)=[f(t)+f(-t)]/2 + 0.5∫g(s)ds (from -t to t)

f is odd, and so is g, so the equation ends up giving zero, as required. But I don`t think that`s what we have to do. How do do I extend the initial conditions so that f(-x)=-f(x)? and the same for g(x). I know how to create an odd function, I can just let h(x)=xf(x2), then h(-x)=-h(x), but I`m not sure what I`m suppose to do.
 
Physics news on Phys.org
anyone?
 
Write doen D'Almberts solution, what do you get?
 
What do you mean? I thought I did write it down,
<br /> u(x,t)=\frac{1}{2}[f(x-t)+f(x+t)]+\frac{1}{2}\int_{x-t}^{x+t} g(s)ds<br />
 
Hm, I think I got it, I`ll try to post it soon.
 
If you've done it, post it otherwise we can go from there. My first thought would be to define two new functions a,b, such that:
<br /> a(x)=\left\{\begin{array}{cc}<br /> f(x) &amp; x\geqslant 0 \\<br /> -f(x) &amp; x&lt;0<br /> \end{array}\right.<br />
Likewise fot b and work from there.
 
Question: A clock's minute hand has length 4 and its hour hand has length 3. What is the distance between the tips at the moment when it is increasing most rapidly?(Putnam Exam Question) Answer: Making assumption that both the hands moves at constant angular velocities, the answer is ## \sqrt{7} .## But don't you think this assumption is somewhat doubtful and wrong?

Similar threads

  • · Replies 11 ·
Replies
11
Views
3K
Replies
6
Views
2K
Replies
5
Views
2K
Replies
2
Views
2K
Replies
4
Views
2K
Replies
3
Views
2K
Replies
3
Views
2K
Replies
6
Views
2K
  • · Replies 2 ·
Replies
2
Views
1K
  • · Replies 11 ·
Replies
11
Views
2K